You are on page 1of 50

BỘ ĐỀ THI HỌC SINH GIỎI TỈNH

ĐỀ HỌC SINH GIỎI VÒNG TỈNH 2008-2009

SECTION 1: LISTENING
I. Listen to the dialogue between Joyce and a woman and then fill in the each blank with ONE word.
You will hear the recording two times.
W: So tell me about your (1) _______________, Joyce.
J: Well, it’s really small town.
W: What is it like there?
J: Oh, I think it’s very (2) _______________ place.
W: Really? Why?
J: Well, there is nothing to do. No good (3) _______________. No nightlife of any kind.
W: Uh, that’s too bad. But small towns are pretty (4) _______________ to live in.
J: Well, yeah, it is (5) _______________ cheap. And lots of people love it because it’s very (6)
_______________.
W: Yeah?
J: Uh-huh. It has great (7) _______________- lots of mountains and rivers, lakes, trees…
W: Well, I don’t know, Joyce. It (8) _______________ like a lovely place!
J: Well, yeah, if you like to go hiking in the summer and (9) _______________ in the winter. But, you
know, I’m not the (10) _______________ type! I’m a real city person.
II. Listen to the letter. Circle the words you hear. You will hear the recording two times.
a. delicious g. warm m. big s. friendly
b. happy h. thin n. loud t. lucky
c. new i. long o. dangerous u. sharp
d. nice j. straight p. safe
e. cold k. jealous q. fair
f. tall l. small r. interesting

SECTION 2: GRAMMAR AND VOCABULARY:


I. Choose the best answer among A, B, C or D to complete each sentence: (10Ms)
1. The technological and economic changes of the 19th century had a marked …………….. on workers
A. cause B. effect C. impact D. consequence
2. The government …………………….. the flood victims with food, clothes and money.
A. gave B. provided C. offered D. presented
3. More and more people …………………….. of food poisoning nowadays
A. exist B. survive C. die D. starve
4. The first sign of vitamin A disorder is night ……………………..
A. loss of sight B. lack of vision C. invisibility D. blindness
5. When you take ice out of the freezer, it ……………………..

1
A. melts B. dissolves C. softens D. disappears
6. There are many …………………….. of pollution in our modern world
A. resources B. sources C. bases D. foundation
7. Many people …………………….. that natural resources will never be use up
A. view B. regard C. consider D. believe
8. Using the computer competently is an important ……………….. to help one get a good job.
A. reason B. aspect C. factor D. issue
9. George wants £1000 for his car, but I don’t think it’s …………….. as much as that.
A. worth B. cost C. valued D. priced
10. “Did Jenny say anything about her sister?”-No, she didn’t …………….. her at all.
A. remind B. remark C. refer D. mention
II. Choose the underlined part ( A, B, C, or D ) that is incorrect: (15Ms)
1. Families who are enough fortunate to own a historic home may be able to get restoration
A B C D
funds from the government.
2. The first things a new international student must do include renting an apartment,
A B
registering for classes, and to get to know the city.
C D
3. When parents allow his children to spend many hours watching television, the children
A B
are not likely to be physically fit.
C D
4. Christianity has become a worldwide religion since it has begun almost two thousand
A B C D
years ago.
5. As the population of Africa continues to grow, animals will continue to lose theirs natural
A B C D
habitat.
6. Actually, by the time Columbus arrived to America in 1492, other Europeans had already
A B C D
reached the New World.
7. The most Americans were killed in World War II than in any other war since the birth of
A B C D
the nation.
8. Antarctica, which is largely covered by ice, receive hardly any rainfall.
A B C D
9. Jessica is only an amateur, but she sings sweeter than most professionals.
A B C D
2
10. Mrs. Adams was surprising that her son and his friend had gone to the mountains to ski.
A B C D
11. A cure for the common cold, causing by a virus, has not been found.
A B C D
12. Amelia Earhart, that was one of the pioneers in aviation, attempted to fly the world in
A B C
1937, but she and her plane mysteriously disappeared over the Pacific Ocean.
D
13. His last test results showed he was the quickest typist, yet he was the less accurate of all the
applicants.
A B C D
14. That secretary of mine is so efficient that she always amazes myself with her speed.
A B C D
15. A problem it has caused setbacks and we have been forced to change the final date of
A B C
completion if we want to avoid being penalized.
D

III. Supply the suitable form of the words in brackets: (10Ms)


Different (1) _________ (conserve) efforts have been made in order to (2) _________ (danger)
species. The Red List – (3) _________ (globe) list of endangered and vulnerable animal species – has
been introduced to raise people’s awareness of conservation needs. Governments have enacted laws to
protect wildlife from commercial trade and (4) _________(hunt). Another _________ (conserve) effort
is the development of wildlife habitat (5) _________ (reserve) as they are suitable for the (6) _________
(survive) of a wide range of species.
The current globe extinction crisis is one of the (7) __________ (great) challenges to mankind. To
preserve the earth’s (8) __________ (value) biodiversity for future generations, the protection of
__________ (danger) species and their habitats should be a top (9) _______ (prior) for international
organizations, (10) __________ (govern) agencies, industry and individuals.
IV. Put the verb in parentheses in correct tenses: (10Ms)
John has always traveled a lot. In fact, he was only two years old when he first flew to the US. His
mother is Italian and his father is American. John was born in France, but his parents 1
(meet)............................ in Cologne, Germany after they 2 (live)…………...... there for five years. They
had met one day while John’s father was reading a book in the library and his mother 3
(sit)……………...... beside him. John 4 (travel)............................ a lot because his parents also travel a
lot. As a matter of fact, John is visiting his parents in France at the moment. He lives in New York now,
but 5 (visit)…….………....... his parents for the last few weeks. He really enjoys living in New York,
but he also loves coming to visit his parents at least once a year. This year he 6 (fly)......................... over
5,000 miles for his job. He has been working for Jackson & Co. for almost two years now. He 7

3
(be)………..…..….... pretty sure that he 8 (work).......................... for them next year as well. His job
requires a lot of travel. In fact, by the end of this year, he 9 (travel)…..……….……..... over 120,000
miles! His next journey will be to Australia. He really doesn’t like going to Australia because it is so far.
This time he 10 (fly)............................. from Paris after a meeting with the company’s French partner.
He will have been sitting for over 18 hours by the time he arrives!
V. Choose the word (A, B, C or D) that best completes each sentence: (25Ms)
1. Do you know _________?
A. who how many people go on Sundays to church
B. who go to church on Sundays how many people
C. how many people who go on Sundays to church
D. how many people who go to church on Sundays
2. We watch the cat ________ the tree.
A. climbed B. climb C. had climbed D. was climbing
3. If we had known your new address, we ________ to see you.
A. came B. will come C. would come D. would have come
4. You may borrow as many books as you like, provided you show them to ________ is at the desk.
A. whoever B. who C. whom D. which
5. He looked forward to ________ his first pay packet.
A. receive B. have received C. be receiving D. receiving
6. Carol refused; ________, her answer was “no”.
A. in other words B. otherwise C. words for words D. however
7. If you see Tom _________ you mind _________ him to get in touch with me?
A. will / reminding B. will / to remind C. would / reminding D. would / to remind
8. _________ I hear that song, I think of you.
A. Whatever B. Forever C. Whenever D. However
9. “Let’s go dancing, _________?” – “Yes, let’s.”
A. will we B. don’t we C. do we D. shall we
10. I wish I _________ all about this matter a week ago.
A. knew B. know C. had known D. B & C are correct.
11. He was _________ he could not wake up.
A. very tired that B. such tired that C. too tired that D. so tired that
12. Joan asked _________.
A. if there was coffee B. there was coffee C. was there coffee D. where was the coffee
13. She didn’t know _________ to have fish or chicken for lunch.
A. both B. whether C. neither D. as
14. For lunch, you may have _________ fish or chicken.
A. both B. neither C. not only D. either
15. I suppose there’s a lot _________ between now and the first show.
A. to be done B. done C. we do D. to have done

4
16. I ____ my house ____. That is why there is all this mess.
A. had – paint B. have – paint C. had had - paint D. am having – painted
17. I warned him _________.
A. to be not late B. don’t be late C. not to be late D. he is not late
18. I should have _________ my hair cut weeks ago, but just don’t seem to have had time.
A. let B. made C. had D. gotten
19. I could not cut the grass because the machine _________ a few days previously.
A. broke down B. has been broken C. had broken down D. breaks down
20. If you want to develop inner tranquility, you have to stop _________ by every little thing that
happens.
A. bothering B. being bothered C. to bother D. to be bothered
21. He was made _________ for two hours.
A. to wait B. wait C. waiting D. waited
22. My boss is angry with me. I didn’t do all the work that I _________ last week.
A. should do B. should have done
C. must have done D. might have done
23. Ann is not at home. She’s __________ to dinner.
A. been B. gone C. went D. being
24. “Where are my jeans?” “They _________ at the moment. Sorry.”
A. are washing B. were washed C. are washed D. are being washed
25. The film was __________. There was so much blood in it.
A. horrify B. horrified C. horrifying D. being horrified

SECTION 3: READING
I. Rearrange the following five paragraphs.
A. One should be wary, however, of assuming that silent reading came about simply because reading
aloud is a distraction to others. Examination that factors related to the historical development of silent
reading reveals that it became the usual mode of reading for most adult reading tasks mainly because the
tasks themselves changed in character.
B. By the end of the century students were being recommended to adopt attitudes to books and to use
skills in reading them which were inappropriate, if not impossible, for the oral reader. The social,
cultural, and technological changes in the century had greatly altered what the term “reading” implied.
C. Reading to oneself is a modern activity which was almost unknown to the scholars of the classical
and medieval worlds, while during the fifteen century the term “reading” undoubtedly meant reading
aloud. Only during the nineteen century did silent reading become commonplace.
D. Towards the end of the century there was still considerable argument over whether books
should be used for information or treated respectfully, and over whether the reading of material such as
newspapers was in some way mentally weakening. Indeed this argument remains with us still in
education. However, whatever its virtues, the old shared literary culture had gone and was replaced by
the printed mass media on the one hand and by books and periodicals for a specialized readership on the
other.
5
E. The last century saw a steady gradual increase in literacy, and thus in the number of readers. As
readers increased, so the number of potential listeners declined, and thus there was some reduction in the
need to read aloud. As reading for the benefit of listeners grew less common, so came the flourishing of
reading as a private activity in such public places as libraries, railway carriages and offices, where
reading aloud would cause distraction to other readers.
Your answers:
1. ------------------ 2. ------------------ 3. ------------------
4. ------------------ 5. ------------------
II. Read the passage and choose the best answer to complete it.
An unlucky day
Helen packed a small suitcase, said goodbye to her mother and hurried out of the house to (1)
__________ to the station. There was no one (2) ____________ waiting at the bus stop, so it looked as
if a bus had just left. Helen looked at her watch anxiously; it was already two o’clock. Her train left at
two thirty, and since it would (3) __________ at least twenty minutes to reach the station, she did not
have much time to (4) __________, even if a bus came along at once.
Just then a taxi came slowly down the road. Helen knew that the (5) __________ to the station
was at least two dollars, which was more than she could (6) ___________, but she quickly made her (7)
___________ that it would be well (8) ___________ the extra expense in order to be sure of catching
the train. So she stopped the taxi and got in. she told the driver that she had to catch the train which left
at half past two. The man nodded and said that he would take a short (9) ___________ to get her to the
station in good time.
All went well until, just as they were coming out of a side street into the (10) _________ that
lead to the station, the taxi ran (11) ___________ a car. There was a loud crash and Helen was thrown
(12) _________ so violently that she hit her head on the front seat. Both drivers got out and began (13)
_________ each other. Helen got out as (14) _____ to ask them to stop quarrelling, but (15) _________
took any notice of her at all.
1. A. go on bus B. get the bus C. drive taxi D. catch the bus
2. A. else B. other C. person D. more
3. A. be B. take C. waste D. lose
4. A. spend B. wait for C. spare D. go
5. A. cost B. expense C. fare D. money
6. A. pay B. afford C. give D. have
7. A. decision B. mind C. head D. own
8. A. cheap B. done C. for D. worth
9. A. time B. way C. cut D. road
10. A. main road B. main side street C. alley D. train
11. A. off B. up C. for D. into
12. A. forward B. over C. up D. hard
13. A. shouting B. to shout at C. to shout for D. shouting to
14. A. well B. soon C. much D. if
15. A. none B. no one C. neither D. both of them

6
III. Read the two passages and choose the best answer to the questions or to complete the statements .
PASSAGE 1:
A pilot cannot fly by sight alone. In many conditions, such as flying at night and landing in dense fog, a
pilot must use radar, an alternative way of navigating. Since human eyes are not very good at
determining speeds of approaching objects, radar can show a pilot how fast nearby planes are moving.
The basic principle of radar is exemplified by what happens when one shouts in a cave. The echo of the
sounds against the walls helps a person determine the size of the cave. With radar, however, the waves
are radio waves instead of sound waves. Radio waves travel at the speed of light, about 300,000
kilometers in one second. A radar set sends out a short burst of radio waves. Then it receives the echoes
produced when the waves bounce off objects. By determining the time it takes for the echoes to return to
the radar set, a trained technician can determine the distance between the radar set and other objects. The
word “radar”, in fact, gets its name from the term “radio detection and ranging”. “Ranging” is the term
for detection of the distance between an object and the radar set. Besides being of critical importance to
pilots, radar is essential for air traffic control, tracking ships at sea, and for tracking weather systems
and storms.
1. What is the main topic of this passage?
A. The nature of radar. B. History of radar.
C. Alternatives to radar. D. Types of ranging.
2. According to the passage, what can radar detect besides location of objects?
A. Shape. B. Size. C. Speed. D. Weight.
3. The word “exemplified” in the passage can be replaced by _______.
A. “specified” B. “resembled” C. “illustrated” D. “justified”
4. The word “shouts” in the passage most closely means _______.
A. “exclaims” B. “yells” C. “shoots” D. “whispers”
5. Which of the following words best describes the tone of this passage?
A. argumentative B. explanatory C. humorous D. imaginative

6. According to the passage, the distance between a radar set and an object can be determined by
_______.
A. the time it takes for a burst of radio waves to produce echoes when the waves bounce off the object
B. the term “ranging” used for detection of the distance between an object and the radar set
C. the time it takes for the radio waves to produce echoes and bounce off the object
D. the time it takes for the echoes produced by the radio waves to return to the radar set
7. Which type of waves does radar use?
A. tidal B. sound C. heat D. radio
8. The word “tracking” in the passage most closely means _______.
A. sending B. searching for C. ranging D. repairing
9. Which of the following would most likely be the topic of the next paragraph?
A. A history of flying. B. Other uses of radar.
C. The technology used by pilots. D. Uses of some technology.
10. What might be inferred about radar?
7
A. It takes the place of a radio.
B. It has improved navigational safety.
C. It was developed from a study of sound waves.
D. It gave birth to the invention of the airplane.
PASSAGE 2:
Today’s cars are smaller, safer, cleaner, and more economical than their predecessors, but the car of the
future will be far more pollution-free than those on the road today. Several new types of automobile
engines have already been developed than run on alternative sources of power, such as electricity,
compressed natural gas, methanol, steam, hydrogen, and propane. Electricity, however, is the only zero-
emission option presently available.
Although electric vehicles will not be truly practical until a powerful, compact battery or other
dependable source of current is available, transport experts foresee a new assortment of electric vehicles
entering everyday life: shorter-range commuter electric cars, three-wheeled neighborhood cars, electric
delivery vans, bikes and trolleys.
As automakers work to develop practical electrical vehicles, urban planners and utility engineers are
focusing on infrastructure systems to support and make the best use of the new cars. Public charging
facilities will need to be as common as today’s gas stations. Public parking spots on the street or in
commercial lots will need to be equipped with devices that allow drivers to charge their batteries while
they stop, dine, or attend a concert. To encourage the use of electric vehicles, the most convenient
parking in transportation centers might be reserved for electric cars.
Planners foresee electric shuttle buses, trains, buses and neighborhood vehicles all meeting at transit
centers that would have facilities for charging and renting. Commuters will be able to rent a variety of
electric cars to suit their needs: light trucks, one-person three-wheelers, small cars, or electric/gasoline
hybrid cars for longer trips, which will no doubt take place on automated freeways capable of handling
five times the number of vehicles that can be carried by freeway today.
1. The following electrical vehicles are all mentioned in the passage EXCEPT
A. vans B. trains C. planes D. trolleys
2. The author’s purpose in the passage is to
A. criticize conventional vehicles
B. support the invention of electric cars
C. narrate a story about alternative energy vehicles
D. describe the possibilities for transportation in the future
3. The passage would most likely be followed by details about
A. automated freeways B. pollution restrictions in the future
C. the neighborhood of the future D. electric shuttle buses
4. The word “compact” in the second paragraph is closest in meaning to
A. long-range B. inexpensive C. concentrated D. squared
5. In the second paragraph, the author implies that
A. a dependable source of electric energy will eventually be developed.
B. everyday life will stay much the same in the future.
C. a single electric vehicle will eventually replace several modes of transportation

8
D. electric vehicles are not practical for the future
6. According to the passage, public parking lots of the future will be
A. more convenient than they are today B. equipped with charging devices
C. much larger than they are today D. as common as today’s gas stations
7. The word “charging” in this passage refers to
A. electricity B. credit cards C. aggression D. lightning
8. The word “foresee” in this passage could best be replaced with
A. count on B. invent C. imagine D. rely on
9. The word “commuters” in paragraph 4 refers to
A. daily travelers B. visitors C. cab drivers D. shoppers
10. The word “hybrid” in paragraph 4 is closest in meaning to
A. combination B. hazardous C. futuristic D. automated

SECTION 4: WRITING
I. Rewrite the sentence as similar in meaning as possible: (20Ms)
1. It is cheaper to go by train than by car.
- Going by car …………………………………………………………………………………
2. We must go now because our train leaves at 8 A.M.
- It’s …………..………………………………………………………………………………
3. Although he made great efforts, he failed to win the race.
- In spite ………………………………………………………………………………………
4. Since she was careless, Mary lost her job.
- Because of …………………………………………..………………………………………
5. Workers are building a new factory.
- A new factory ………………………………………….……………………………………
6. Who broke the window?
- By ……………………………………………………………………………………………
7. “What’s your job?” Mary asked Tom.
- Mary wanted to know ………………………………………………………………………
8. The bust couldn’t run because of the fog.
- The fog ………………………………………………..…………………………………….
9. Tim will be eighteen next week.
- It is Tim’s ……………………………………………………………………………………
10. Peter said to Paul, “I didn’t use your mobile phone! Someone else did, not me”.
- Peter denied …………………………………………………………………………………
II. Write a report of about 150 words describing the information given in the chart. Choose only ONE
chart (A or B) to write (20Ms)

A. The chart below illustrates heart attacks by age and gender in the USA.
9
B. The chart below shows the amount of time an average student spends on various subjects.
Study the chart carefully and comment on the proportion of time they spend on these subjects.

50
45
40
35 Natural
30 sciences
25 English
20
15
Social
10
sciences
5
0
1995 2000 2005 2007
……
………………………………………………………………………………………………………………
………………………………………………………………………………
………………………………………………………………………………………………………………
……………………………………………………………………………………
………………………………………………………………………………………………………………
……………………………………………………………………………………
………………………………………………………………………………………………………………
……………………………………………………………………………………
………………………………………………………………………………………………………………
……………………………………………………………………………………

10
………………………………………………………………………………………………………………
……………………………………………………………………………………
………………………………………………………………………………………………………………
……………………………………………………………………………………
………………………………………………………………………………………………………………
……………………………………………………………………………………
………………………………………………………………………………………………………………
……………………………………………………………………………………
………………………………………………………………………………………………………………
……………………………………………………………………………………
………………………………………………………………………………………………………………
……………………………………………………………………..……………
………………………………………………………………………………………………………………
……………………………………………………………………………………
………………………………………………………………………………………………………………
……………………………………………………………………………………
………………………………………………………………………………………………………………
……………………………………………………………………………………
………………………………………………………………………………………………………………
……………………………………………………………………………………
………………………………………………………………………………………………………………
……………………………………………………………………………………
III. Make all the changes and additions necessary, to produce from the following sets of words or
phrases, sentences that together make a complete letter: (20Ms)
Dear Mary,
1. Thank you/ much/ your letter/ arrive/ few days ago.
- .......................................................................................………………………………………..
2. It/ be lovely/ hear/ you.
- ……………………………………………………………………………………………….
3. I/ be sorry/ I not write/ such/ long time/ but I/ be/ very busy.
- .......................................................................................…………………………………………
4. As you know/ we buy/ new house/ May.
- .......................................................................................……………………..………………….
5. It/ be/ very bad condition/ and it need/ a lot/ work.
- .......................................................................................………………………………………….
6. We finish/ most/ it now/ and it look/ very nice.
- .......................................................................................……………………………………….
7. John and I/ decide/ give/ house – warming party/ October 2nd.
- .......................................................................................………………………………………..
8. You thinks/ you/ able/ come?

11
- .......................................................................................…………………………………………
9. Please give me/ ring/ let/ know/ you/ make it.
- .......................................................................................………………………………………..
10. I/ really/ look forward/ see you again.
- .......................................................................................………………………………………..
Love,
Bob

THE END

12
Đáp án 2008-2009:

SECTION 1: LISTENING
I. Listen to the dialogue between Joyce and a woman and then fill in the each blank with ONE word.
You will hear the recording two times.
(1) hometown (6) pretty
(2) boring (7) scenery
(3) restaurants (8) sounds
(4) inexpensive (9) skiing
(5) fairly (10) outdoors
II. Listen to the letter. Circle the words you hear. You will hear the recording two times.
a, b, c, d, f, h, i, j, l, m, n, p, r, s, t
Dear Gramma Lulu,
Thanks so much for the cookies. They’re delicious. I’m happy here at Oregon State. My roommate,
Yoko, is from Japan. He’s a new student, too. She’s very nice. She’s tall and thin. Her hair is long
and straight.
We have a small apartment and a big dog. Our dog’s name is Rocky. He’s big and loud. We’re safe
with Rocky around.
My classes are interesting and my teachers are friendly. I’m lucky to be here.
I hope you’re fine. Please write.
Love,
Carol

SECTION 2: GRAMMAR AND VOCABULARY:


I. Choose the best answer among A, B, C or D to complete each sentence:
1. C 6. B
2. B 7. D
3. C 8. C
4. D 9. A
5. A 10. D
II. Choose the underlined part ( A, B, C, or D ) that is incorrect.
1. B 9. C
2. D 10. A
3. A 11. B
4. C 12. A
5. D 13. A
6. C 14. C
7. A 15. A
8. B

13
III. Supply the suitable form of the words in brackets:
1. conservation 6. survival
2. endangered 7. greatest
3. global 8. valuable
4. overhunting 9. priority
5. reserves 10. governmental.
IV. Put the verb in parentheses in correct tenses.
1. had met 6. has flown
2. had been living 7. is
3. sat 8. will be working
4. travels 9. will have traveled
5. is visiting 10. is going to fly
V. Choose the word (A, B, C or D) that best completes each sentence.
1. D 14. D
2. B 15. A
3. D 16. C
4. A 17. C
5. D 18. C
6. A 19. C
7. C 20. B
8. C 21. A
9. D 22. C
10. C 23. B
11. D 24. D
12. A 25. C
13. B

SECTION 3: READING
I. Rearrange the following five paragraphs.
1. C
2. A
3. E
4. D
5. B
II. Read the passage and choose the best answer to complete it.
1. D 5. B
2. A 6. B
3. B 7. D
4. C 8. C
14
9. A 13. B
10. D 14. A
11. A 15. C
12. B
III. Read the two passages and choose the best answer to the questions or to complete the
statements .
PASSAGE 1:
1. A 6. D
2. C 7. D
3. C 8. B
4. B 9. B
5. B 10. B
PASSAGE 2:
1. C
2. D
3. A
4. C
5. A
6. B
7. A
8. C
9. A
10. A

15
D. WRITING:
I. Rewrite the sentence as similar in meaning as possible:
1. Going by car is more expensive than (going) by train
2. It’s time we went because our train leaves at 8 A.M
3. In spite of his great efforts, he failed to win the race
4. Because of her carelessness, Mary lost her job
5. A new factory is being built
6. By whom was the window broken?
7. Mary wanted to know what Tom’s job was
8. The fog prevented/ stopped the bus from running
9. It is Tim’ eighteenth birthday next week
10. Peter denied using/ having used Paul’s mobile phone (and said that some else had done)
II. Write a report of about 150 words describing the information given in the chart. (2ms)
Describing a chart - 150 words
+ Topic sentence : 0.25 m
+ Supporting sentences : 1.5 ms
+ Concluding sentence : 0.25 m
III. Make all the changes and additions necessary, to produce from the following sets of words or
phrases, sentences that together make a complete letter: (20Ms)
Dear Mary,
1. Thank you very much for your letter which arrived a few days ago
2. It was lovely to hear from you
3. I am sorry I haven’t written for such a long time but I’ve been very busy
4. As you know, we bought a new house in May
5. It was in very bad condition and it needed a lot of work
6. We have finished most of it now and it looks very nice
7. John and I have decided to give a house –warming party on October 2nd
8. Do you think you will be able to come?
9. Please give me a ring and let me know if you can make it
10. I’m really looking forward to seeing you again
Love
Bob

Së GD-§T Hµ Néi §Ò THI CHäN §éi tuyÓn häc sinh giái líp 12
Tr-êng THPT Xu©n Mai N¨m: 2008-2009
---------- M«n: TiÕng Anh
Thêi gian lµm bµi: 150 phót (kh«ng kÓ thêi gian ph¸t ®Ò)
(§Ò thi gåm 5 trang – ThÝ sinh lµm bµi vµo ®Ò thi)

16
Hä vµ tªn:…………………………………………………….. Líp:……………………….

PART I. Tr¾c nghiÖm

I. PHONETICS. Choose the word whose stress pattern is different from that of the others.
1. A. weather B. unique C. highland D. ladder
2. A. experience B. hurricane C. volcano D. material
3. A. dependence B. prepare C. prediction D. forecast
4. A. tidal B. ancient C. amount D. heavy
5. A. tropical B. commercial C. disatrous D. extensive

II. VOCABULARY. Choose the best option (A, B, C or D) which can best replace the underlined
word/words.
6. Germination of seeds begins with the absorption of water, the swelling of the whole seed, and the
cracking of the seed coat.
A. outer B. entire C. wet D. stiff
7. The object is to make our reseearch readable and understandable.
A. comprehension B. comprehensible C. comprehensive D. comprehend
8. The home of Helen Keller was a crossroads where people gathered to discuss art, politics, and the state
of the world.
A. huge rural estate B. national monument C. central meeting place D. quiet refuge
9. The American Medical Assiciation has called for the sport of boxing to be banned.
A. forbidden B. regulated C. reorganised D. encouraged
10. The Litlle Foxes, a drama by Lillian Hellman, was first introduced in New York in 1939.
A. play B. novel C. music D. dance
11. The policy of separation of church and state has limited direct government support of private schools in
the United States.
A. restricted B. reversed C. demanded D. imposed
12. In the twentieth century, drug markedly improved health throughout the world.
A. recently B. consistently C. supposedly D. noticeably
13. We finally managed to get her to join us in our games.
A. persuade her B. convince her C. insist her D. A and B
14. Their attempt to gain a seat in the Parliament came to a success in the end.
A. showed off B. went off C. came off D. broke off
15. The most dramatic example of static electricity is lightning.
A. examination B. distance C. instance D. purpose
16. Surveying is the science of calculating exact distances and directions between points of the Earth’s
surface.
A. precise B. local C. obscure D. great

17
17. Bone and ivory are light, strong, and accessible materials for Inuit artists.
A. available B. beautiful C. economic D. natural
18. Ants follow scent trails so precisely they can locate their nests without hesitation.
A. build B. find C. clean D. fill
19. Although its takeoff in April in 1981 was besets with delays, the space shuttle Columbia returned
triumphantly two days later from the its mission in the Earth’s orbit.
A. staff B. rescue C. funding D. launching
th
20. In the first decades of the 20 century,the individual gene could not be seen, but could be worked with
fruitfully.
A. blindly B. completely C. productively D. carefully
III. GRAMMAR AND STRUCTURES. Choose the most suitable answer.
21. “Did you finish the report?”. – Yes, but I wish I had known…………………….
A. how hard it would be B. how hard would it be C. how would it be hard D. how would it be
22. “Why did Bob get into trouble?” – “Someone heard him………………..his views of the boss”.
A. to express B. was expressing C. expressed D. expressing
23. If you want to join the History Society, you must……………..this application form.
A. wake up B. fill in C. write down D. do up
24. “This traffic is terrible”, she said. “You’re right. People should……………..”.
A. less use their cars B. use less their cars C. use their cars less D. their cars use less
25. “Computers certainly have changed the world”. - “Yes, they make……………..to store information”.
A. it easier for people B. it more easily for C. people easier D. easier for people
26. “Are your parents coming to your concert?” – “Yes, but they seem more nervous about…….playing”.
A. watching me than B. watching me C. watching than I am about D. watch me
27. “I want to buy that car”. - “…………… its good qualities, it has one drawback”.
A. Although B. Whether C. However D. Despite
28. ……………the students here are serious about studying.
A. Most B. Almost C. Almost of D. Most of
29. “Which is more important, luck or effort?” - “ Luck is…………….effort”
A. of the same importance B. the same importance as
C. of the same importance as D. as the same important as
30. He drives so fast that I am afraid one day he will…………somebody crossing the street.
A. crash down B. knock down C. turn over D. run
across

IV. READING.
A. Read the following passage and answer the questions by choosing A, B, C or D
1st passage
NOTICE: To all patrons of the Blue Wave Fitness Center
The management would like members to note that the Blue Wave Fitness Center is not liable for any items
left in the lockers. Should you lose anything from the locker, we regret that we will be unable to assist you

18
in any way. Therefore, it is important that you make certain that your locker is kept locked while you are
using our facilites. Also, for security reasons, we do not keep a record of members’s locker combination
code, so it is vital that you remember the combination you choose.
31. Where would this notice be posted?
A. In a changing room B. At a railway station C. In a restroom D. In an office
32. What do members need to open their lockers?
A. a series of numbers B. coins C. keys D. Nothing
33. Why do you think this notice was posted?
A. The Blue Wave has suffered from thefts recently B. The Blue Wave has bought new lockers.
C. The Blue Wave changed its management. D. The Blue Wave is looking for new members.
2nd passage
Sometimes you know things about people the first time you see them, for example, that you want to be
friends with them or that you don't trust them. But perhaps this kind of intuition isn't as hard to explain as it
may seem. For instance, people give out body language signals all the time. The way you hold your body,
head and arms tells people about your mood. If you hold your arms tightly at your sides, or fold them
across your chest, people will generally feel that you are being defensive. Holding your head to one side
shows interest in the other, while an easy, open posture indicates that you are self-confident. All this
affects the way you feel about someone.
Also, a stranger may remind you of a meeting with someone. This may be because of something as simple
as the fact that he or she is physically similar to someone who treated you well or badly. Your feelings
about a stranger could be influenced by a smell in the air that brings to mind a place where you were happy
as a child. Since even a single word can bring back a memory such as that, you may never realize it is
happening.
34. What does the word "open" in the passage most closely mean?
A. Unrestrained. B. Relaxed. C. Confined. D. Unlimited. 35.
What influences your impression of a person you meet the first time?
A. Intuition. B. Familiarity. C. Knowledge. D. Feeling.
36. What one feels about a stranger may be influenced by something that ________.
A. strengthens one's past behaviours B. reminds one of one's past treatment
C. revives one's past memories D. points to one's childhood
37. What does the second paragraph discuss?
A. Meanings of signals one implies towards a stranger.
B. Factors that may influence one's feelings about a stranger.
C. How people usually behave to a stranger.
D. Factors that cause people to act differently.
38. Intuition described in the passage can be explained by means of _________.
A. styles B. languages C. patterns D. behaviours

B. Read the passage, then choose one word (A, B, C or D) to fill in each blank.
MENTORING
Many adults in America and increasing numbers elsewhere(39)……part in mentoring schemes. A mentor
is an adult who provides support and friendship to a young person. There are(40)……different ways of

19
mentoring: passing on skills, sharing experiences, offering guidance. Sometimes the most(41)…...thing to
do is just listen. Mentoring is open to anybody – no particular(42)……..experience required, just a desire to
make a(43)……to the life of a young people who needs help. This may seem a difficult thing at first, but
many people find they have a real(44)……….for it.
The support of a mentor can play an important(45)………in a child’s development and can often make
up(46)…………a lack of guidance in a young person’s life. It can also improve young people’s(47)……..
towards society and build up their confidence in dealing with life’s challenges. For the mentor, it can be
incredibly rewarding to know that they have had a significant(48)………..on a child and helped to give
them the best chance in life.(49)………,it is not only adults who are(50)…………..of taking on this role.
There is now an increasing (51)………for teenagers to mentor young children, for example by helping
them with reading or other school work.
39. A. hold B. give C. take D. form
40. A. number B. numerous C. double D. massive
41. A. helpful B. willing C. kind D. unlikely
42. A. trained B. difficult C. professional D. skilled
43. A. home B. switch C. difference D. distance
44. A. skill B. strength C. talent D. ability
45. A. piece B. section C. group D. part
46. A. to B. for C. with D. over
47. A. attitude B. impression C. agreement D. conduct
48. A. desire B. consequence C. experience D. influence
49. A. However B. Whenever C. Although D. In deed
50. A. able B. capable C. possible D. efficient
51. A. want B. wish C. demand D. lack

V. WRITING. Choose A , B, C or D whose underlined part is not grammatically right.


52. The song to that we listened last night was beautiful
A B C D
53. He gave me the pen in which I wrote my lessons.
A B C D
54. Can you explain what the numbers of these animals have become small?
A B C D
55. The sun releases large numbers of energy every day.
A B C D
56. The energy of the sun can be changed to electricity.
A B C D
PART II. Tù luËn

I. READING
A. Supply the correct forms of the words in brackets to complete the passage.
MEN AND WOMEN ARE NOT EQUAL HERE!

20
There is a big (57. DIFFER)………………………in the Bijago Islands off the west coast of Africa in the
roles that men and women play. For example, men look after the children and wear jewlry and perfume
when they go out. Women find building houses (58. PREFER)…………………………to shopping and do
all kinds of job which men would normally do in other countries. If they want a husband, they ask his
mother for (59. PERMIT)…………………….and do not need to obtain his (60. AGREE)………………..
when they plan their (61. MARRY)…………………….to him. Most men spend hours every day standing
in front of a mirror bushing their hair and choosing nice clothes to wear.
“I don’t want to make any (62. CRITIC)……………………….about the women in our society, but I think
it’s time that sexual (63. DISCRIMINATE)………………………….against men stopped”, one man said.
“Some visitors to our islands find our customs very (64. AMUSE)…………………..but I feel very (65.
NERVE)…………………………….when any girl visits my mother”, another man said.

B. Complete the following passage by filling each blank with one suitable word.
In Western countries, electricity, gas, and water are not luxuries but necessities. Companies now realise
that consumers want products that will not (66)………….…work effectively, but save money as well.
For most North American households, lighting accounts for 10% to 15% of the electricity bill. However,
this amount can be (67)…………………….by replacing an old ordinary 100-watt bulb with an energy-
saving one. These bulbs use a quarter of the electricity of standard (68)……………………, and last eight
times longer. Therefore consumers can (69)………………..about US$7 to US$21 per bulb.
In Europe, there is a labeling scheme for fridges, freezers, washing machines and tumble dryers. The lable
tells the consumers how (70)………………….energy effeciency each model has, compared with other
appliances in the same category.

II. WRITING
A. Rewrite the following sentences beginning with word/words or phrases provided.
71. He tries hard but he cannot master English pronunciation.
 No matter..….………………………………………………………………………………………….
72. Wherever he goes, his mother goes too.
 His mother….……………………………………………………………………………..….……….
73. The garage is going to repair the car for us next week.
 We……..………………………………………………………………………………….….……….
74. It’s your decision whether we stay in or go out.
 It’s up….……………………………………………………………………………………...……….
75. No explanation is needed.
 It is……..……………………………………………………………………………………..……….
76. I can meet you if you arrive before eleven.
 So……………………………………………………………………………………………….….….
77. He would prefer you to pay him immediately.
 He’d rather..……………………………………………………………………………………..…….
78. Nobody can deny that he has a beautiful voice.
 It can’t….………………………………………………………………………………………..…….
79. The furniture was so expensive that I didn’t buy it.

21
 It was so……………………………………………………………………………………………….
80. He didn’t arrive in England untill the end of December.
 It wasn’t……………………………………………………………………………………………….

B. Use the word given and other words to complete the second sentence so that it has similar meaning
to the first. Do not change the word given.
81. Tom shares a lot of the same things with Ken. (common)
 Tom and Ken…..…………………………………………………………………………..………….
82. Their house has been broken into three times this year. (have)
 They…………………………………………………………………………………………………..
83. I hope you enjoy yourself while you stay with Mabel. (during)
 I hope you…..…………………………………………………………………………………...……
84. He looks remarkably like his father. (resemblance)
 He bears.…………………………………………………………………………………………….
85. I started studying law in 2003, and now I’m still studying law. (since)
 I have….…………………………………………………………………………………………….
86. I am sorry that I have caused you so much trouble. (apologize)
 I………..…………………………………………………………………………………………….
87. Jill doesn’t often walk to the college. (unusual)
 It………..……………………………………………………………………………………………
88. My sister always talks when I am watching my favourite programme. (habit)
 My sister….………………………………………………………………………………………….
89. Our team won and the visitors lost. (beaten)
 The visitors.………………………………………………………………………………………….
90. The number of road accidents has increased drammatically recently. (drammatic)
 There…..…………………………………………………………………………………………….

C. Write a passage of about 100-120 words on the benefits of the Internet. Your passage must cover
the following:
- The Internet as a source of information.
- The Internet as a source of entertainment.
- The Internet as a source of education.
Your writing.
…………………………………………………………………………………………………………..
…………………………………………………………………………………………………………..
…………………………………………………………………………………………………………..
…………………………………………………………………………………………………………..
…………………………………………………………………………………………………………..
…………………………………………………………………………………………………………..
…………………………………………………………………………………………………………..

22
…………………………………………………………………………………………………………..
…………………………………………………………………………………………………………..
…………………………………………………………………………………………………………..
…………………………………………………………………………………………………………..
…………………………………………………………………………………………………………..
…………………………………………………………………………………………………………..
…………………………………………………………………………………………………………..
…………………………………………………………………………………………………………..
…………………………………………………………………………………………………………..
…………………………………………………………………………………………………………..
…………………………………………………………………………………………………………..
…………………………………………………………………………………………………………..
…………………………………………………………………………………………………………..
…………………………………………………………………………………………………………..
…………………………………………………………………………………………………………..
…………………………………………………………………………………………………………..
…………………………………………………………………………………………………………..
…………………………………………………………………………………………………………..
…………………………………………………………………………………………………………..
…………………………………………………………………………………………………………..
…………………………………………………………………………………………………………..
…………………………………………………………………………………………………………..
…………………………………………………………………………………………………………..
…………………………………………………………………………………………………………..
………………………………………………………………………………………

§Ò luyÖn thi HSG sè 8


(School year: 2007-2008)
Question I: Pick out the word with the underlined part pronounced differently from the rest.
1. A.match B. market C. tap D. cap
2. A.book B. look C. floor D. hook
3. A.hire B. find C. file D. film
4. A.hate B. fat C. cat D. bat
5. A.sun B. pull C. fun D. hunt
6. A.pardon B. hard C. parent D. park
7. A.rose B. cover C. nose D. over
8. A.stage B. manage C. village D. baggage
9. A.supply B. supermaket C. support D. suppose
10. A.blood B. mood C. moon D. soon
23
Question II: Chooe the word or phrase (A, B, C or D) which best completes each sentence.
1. " I thought that the tour began at 3:00.
"Oh no, you're .................... It began at 1:30 "
A. too much late here C. here too much late
B. here much too late D. too much here late
2. Although he is intelligent, he doesn't do well at school.
A. Despite being intelligent, he doesn't do well at school.
B. Although his intelligence, he does well at school.
C. In spite of intelligent, he doesn't do well at school.
D. In spite he is intelligent, he doesn't do well at school.
3. Jill drives more carefully than Rose.
A. Rose drives less careful than Rose. B. Jill is a more careful driver than Rose.
C. Rose drives carelessly than Rose. D. Jill is a more carefully driver than Rose.
4. Jim is five centimeters _____ than Tom.
A. higher B. tall C. taller D. tallest
5. I asked her ___________ she understood the lesson.
A. if not B. if C. even if D. if only
6. The man ________we met yesterday was the manager of a bicycle factory.
A. whose B. which C. who D. when
7. London is ___________ of England.
A. one capital B. a capital C. capital D. the capital
8. I don't believe a word she says, I think she ____________.
A. is laying B. is telling lie C. is lying D. lied
9. AIDS is a(n) ____________disease.
A. endanger B. dangerous C. danger D. endangered
10. I knew they were talking about me ________ they stopped when I entered the room.
A. so that B. therefore C. because D. despite
11. You will have to work hard if you want to ___________.
A. successful B. succeed C. successfully D. success
12. She has worked as a secretary _____ she graduated from college.
A. since B. until C. while D. before
13. When I came to visit her last night, she _____ a bath.
A. was having B. has C. is having D. had
14. He has really worked hard so far, ____________ he?
A. hasn't B. has C. doesn't D. does
15. He is learning English ___________ he can study in England.
A. so as to B. so as C. in order to D. so that
16. English ___________ in many parts of the world.
A. is spoken B. is speaking C. was spoken D. speaks
17. The children ___________ to the zoo.
24
A. were enjoyed taking B. were enjoyed taken
C. enjoyed taking D. enjoyed being taken
18. My parents first _____ each other at the Olympic Games in 1982.
A. meet B. met C. had meet D. have met
19. The following day she felt ________ well to go to work.
A. sufficient B. sufficiency C. suffice D. sufficiently
20. ____________ he comes in half an hour, we shall go alone.
A. If B. Unless C. Because D. When
21. Whether we go out will depend________ the weather.
A. about B. with C. in D. on
22. Their ___________ has lasted for more than 20 years.
A. friends B. friendship C. friend D. friendly
20. If I had time, I ____________ to the beach with you this weekend.
A. would have gone B. would go C. will have gone D. will go
24. She failed the test, ___________she studied hard.
A. despite B. as though C. as D. although
25. He took his seat quietly _____________.
A. in order that not to disturb their conversation
B. in order not disturb their conversation
C. so as not to disturb their conversation
D. so as to disturb their conversation

Question III: Read the text below. Use the words given in capitals at the end of each line to form a
word that fits in the space in the same line
My ideal job
One thing I know is that I wouldn't like to have an occupation OCCUPY
that has anything to do with physics, (1) .................................. or maths; CHEMIST
I am not the (2) .................................. type at all. In fact at school, I was a SCIENCE
complete (3) .................................. in these subjects. Neither am I very FAIL
good at dealing with people, nor am I (4) .................................., so jobs in AMBITION
business, administration and (5) .................................. don't really interest MANAGE
me either. Moreover I find it (6) .................................. to be surrounded by IRRITATE
a lot of people; I would much rather have a job involving creative
work or (7) .................................. skills of some sort. I'd like to have the ART
chance to work outdoors (8) .................................. and perhaps do a bit of OCCASION
travelling too. I am not (9) .................................. concerned about becoming PARTICULAR
rich but I would like to have a (10) .................................. income - enough REASON
to live comfortably.
Question IV: Read the following passage and choose the most suitable word for each space.

25
According to a magazine article I read recently, we (1) ......... live in an age of increasing leisure.
Not only are more and more people reaching (2) ......... age with their taste for enjoyment and even
adventure relatively (3) ......... but the working week is becoming shorter and the opportunities for (4) .........
are becoming greater and greater all the time. Not to mention the fact that people (5) .........to spend less
time travelling to work or may even be working from home. What I can't understand, however, is who
these people are. As far as I can (6) ......... the whole thing is another one of (7) ......... journalistic fictions. I
admit that there are a lot of retired people (8) ......... but I am not sure that all of them are dashing about
learning hang-gliding or sailing single-handed (9) ......... the world. My own parents seem to (10) .........
most of their time gazing at the television. And as for the shorter working week, I wish someone (11)
.........remind my company about it. I seem to be working longer and longer hours (12) ......... the time. The
little leisure time I have is eaten into by sitting in the traffic jams or waiting for trains to (13) ......... up at
rain-swept platforms. I haven't noticed any dramatic improvements in my (14) ......... either, but perhaps I
just have to wait until I get my (15) .........

1. A. presently B. at the moment C. now D. at this time


2. A. retirement B. their C. later D. third
3. A. present B. survived C. free D. intact
4. A. this B. longer C. leisure D. people
5. A. use B. tend C. have D. demand
6. A. concern B. imagine C. expect D. tell
7. A. the B. those C. these D. some
8. A. in our days B. in these times C. nowadays D. now and again
9. A. round B. over C. through D. into
10. A. have B. use C. the D. spend
11. A. would B. to C. had D. might
12. A. at B. for C. take D. all
13. A. keep B. line C. show D. set
14. A. cost of living B. lifestyle C. lifeline D. livelihood
15. A. pension B. retirement C. insurance D. salary
Question V: Complete the sentence with the most appropriate word in the box.

born music audience deal


called areas mixture on
began channel broadcast English live

MTV stands for Music Television. It's a television (1).................. dedicated to pop music. It was
(2)................ on 1st August 1981 in the United States. Because of MTV's instant success in the US, the
company expanded into other (3)................... . MTV Europe (4)............... operating on 1st August 1987.
MTV Europe(5) .................. 24 hours a day from its London studios. It can be seen in 33 countries and
reaches an estimated (6)...................... of 110 million viewers.
People of 19 different nationalities work at London headquarters, and they try to offer a (7)
.................. of music from all over Europe. The channel broadcasts in (8)................... but Germany provides
the biggest number of viewers. Currently, one fifth of the (9)................ is by German artists.

26
Most of TV output is video and concerts, but there is also a programme (10).................. Unplugged,
where major artists play (11) ...................... and acoustic in front of a small studio audience.
In addition to music, the channel's programmes (12)..................... with news, movie information and
comedy. MTV has also broadcast special report (13)................... racism, immigration and unemployed
teenagers.

Question VI: Rewrite each sentence so that it contains the word in capitals and so that the meaning
stays the same.
1. Everyone thought the book was written by the princess herself.
 The princess was ...............................................................................................................
2. I can't lift this table on my own.
 I can't lift this table unless ................................................................................................
3. The coins are believed to have been buried for safe-keeping.
 It is believed that ..............................................................................................................
4. They suspended Jackson for the next two matches.
 Jackson was ......................................................................................................................
5. I like cheese best for breakfast.
 What I like ........................................................................................................................
6. It's very important for me to know the answer.
 To know the answer...........................................................................................................
7. Why don't we go to the cinema tonight?
 Let's ...................................................................................................................................
8. One of my favourite pastimes is watching plays.
 Going ................................................................................................................................
9. Americans are usually quite self-confident.
 People in the United States ...............................................................................................
10. Children don't attend classes on Saturday.
 Children don't ...................................................................................................................
11. Andrew said the mistake was my fault.
 Andrew blamed me ...........................................................................................................
12. It was raining all evening.
 It didn't...............................................................................................................................
13. When he got to the party everyone was chatting and eating.
 On his arrival.....................................................................................................................
14. They ate dinner and discussed the problem at the same time.
 While.................................................................................................................................
15. Cats are not as loyal as dogs.
 Dogs .................................................................................................................................
Question VII: In an essay of about 200 words, name the harmful as well as the beneficial effects of
advertising.

27
............................................................................................................................................................................
...............
............................................................................................................................................................................
...............
............................................................................................................................................................................
...............
............................................................................................................................................................................
...............
............................................................................................................................................................................
...............
............................................................................................................................................................................
...............
............................................................................................................................................................................
...............
............................................................................................................................................................................
...............
............................................................................................................................................................................
...............
............................................................................................................................................................................
...............
............................................................................................................................................................................
...............
............................................................................................................................................................................
...............
............................................................................................................................................................................
...............

28
Keys answers:
Question V: Complete the sentence with the most appropriate word in the box.
1. channel 5. broadcast 8. English 11. live
2. born 6. audience 9. music 12. deal
3. areas 7. mixture 10. called 13. on
4. began/ started
Question IV: Complete the sentence with the most appropriate word in the box.

Q# 1 2 3 4 5 6 7 8 9 10 11 12 13 14 15
Key C A D C B D B C A D A D C B A

29
SỞ GIÁO DỤC VÀ ĐÀO TẠO KỲ THI CHỌN HỌC SINH GIỎI TỈNH
THỪA THIÊN - HUẾ LỚP 12 THPT NĂM HỌC 2003 - 2004
*** ------------------------------------------------------
ĐỀ CHÍNH THỨC Môn: TIẾNG ANH - (Vòng 2)
---------------------------------------------------------------------------------------------------------------------
HƯỚNG DẪN CHẤM

II. PHONOLOGY (10 points)


Q# 1 2 3 4 5 6 7 8 9 10
Key A C D B D C D D C A

III. VOCABULARY & GRAMMAR: (20 points )


Part 1. (10 points)

Q# 1 2 3 4 5 6 7 8 9 10
Key B D D D D C A C D B

Part 2. (10 points)


1. put me up 5. putting them across 9. bring her round
2. catch me out 6. bring it up 10. held me up.
3. made it up 7. pass it on
4. get it over 8. drop them off
IV. READING (20 points)
Part 1. (7 points)
Q# 1 2 3 4 5 6 7
Key D B B C D B D

Part 2. (13 points)


V. WRITING: (30 points)

Part 1. (10 points)


1. The princess was thought to have written the book herself.
2. I can't lift this table unless you help me.
4. It is believed that the coins were buried for safe-keeping.
5. Jackson was banned (from playing) for the next two matches.
7. What I like best for breakfast is cheese.
8. Under no circumstances are you to leave the hospital.

30
Part 2. (20 points)
The marks of the essay : 20
(Markers are requested to use the Marking Scheme for writing tasks)

Total marks : 100

---------------------------------------

UBND TỈNH THỪA THIÊN HUẾ KỲ THI CHỌN HỌC SINH GIỎI TỈNH
SỞ GIÁO DỤC VÀ ĐÀO TẠO LỚP 12 THPT NĂM HỌC 2004 - 2005
*** ------------------------------------------------------
ĐỀ CHÍNH THỨC Môn: TIẾNG ANH - (Vòng 2)
Thời gian: 150 phút (không kể thời gian giao đề)
ĐIỂM NHẬN XÉT PHÁCH

........................
............................

---------------------------------------------------------------------------------------------------------------------
I. LISTENING. (20 points)
Part 1: Listen to the phone-in. Number the opinions in the order that you hear them. Which two opinions
are not expressed. You will hear the recording twice.
A. It is difficult to control what children watch.
B. Children can tell the difference between fantasy and reality.
C. Television should show real life.
D. people should be allowed to watch anything they want.
E. There's too much bad language on TV.
F. There's too much unnecessary violence on TV.
G. Only sick people watch violence film.
H. People blame TV for violence so that they can censor it.
I. Children naturally act out scenes from films and TV programmes.
K. Children need to learn that life isn't always pleasant.

Answers:
1......... 2......... 3. ......... 4. ......... 5. ......... 6. ......... 7. .........
8. .........
The two opinions not expressed are: 9. ......... 10.........

31
Part 2: Listen to five situations and complete the table. You will hear the recording twice.
Situation Problem Action offered Choice
1 replacement or refund?
2 smoked salmon
3 left gloves behind
4 put in new battery?
5 dented car

32
III. VOCABULARY & GRAMMAR (25 points)
Part 1: Choose the best answer to complete each sentence. Circle the letter next to the answer you choose.
1. These figures give you some idea of the cost of ….. your car for one year.
A. controlling B. handling C. managing D. maintaining
2. It can take up to three months to ….. a man to do this specialist work.
A. guide B. raise C. train D. learn
3. In today’s paper it ….. that we shall have an election this year.
A. says B. admits C. expresses D. proposes
4. I had to pay ….. on a carpet I bought in through the Customs today.
A. taxes B. rates C. fines D. duty
5. Tropical diseases are comparatively ….. in Europe.
A. scarce B. rare C. slight D. few
6. The ….. charged by the architect for the plans of the new building were unusually high.
A. hire B. price C. fees D. sum
7. It takes a great deal of ….. for the class to make a trip abroad.
A. arrangement B. organisation C. expense D. business
8. You shouldn’t eat so many sweets. They’re ….. for you.
A. bad B. unhealthy C. unsuitable D. disagreeable
9. I wondered whether you would like to ….. to the theatre tomorrow.
A. visit B. go away C. go out D. walk out
10. Do you think he is ….. of doing the job?
A. capable B. competent C. able D. suited

Part 2: Use the correct forms of the words in the brackets to complete the passage below.
Centuries ago, when gold coins (1) (exclusive) ……………………….. were used as the money of
nations and city-states, the (2) (compare) ………………………. value of each nation’s money was (3)
(determine) ………………………... by the ratio of gold content of each coin. Today, gold coins are “used”
only by (4) (collect)………………………. . (5) (nation) ………………………… money is printed on
paper. Each country has its own currency, with names such as pound, dollar, dong, yuan and so on. (6)
(decide) ……………………….. the rate for the international exchange of money is one of the most
complex, and to many (7) (observe) …………………………, one of the most (8) (fascinate)
…………………………. aspects of international banking. Each major currency has a “par value” that is
(9) (usual) ………………... defined, (10) (office) ………………………., in terms of gold.

Part 3: The following sentences are badly constructed. Rewrite them in better style and correct any
grammatical errors.
1. All students must pay their fees, except foreigners, to the university bursar.
...........................................................................................................................................................
2. Swimming is a sport to any healthy person I would recommend.
...........................................................................................................................................................

33
3. The president said that the new education programme was essential addressing a large audience in the
provincial capital yesterday.
...........................................................................................................................................................
4. Talks have begun to plan a railway linking the provincial capitals between the representatives of the
provincial assemblies.
...........................................................................................................................................................
5. The advertisement said that they wished to employ a secretary for an expanding company with good
shorthand and typing speeds at their head office.
...........................................................................................................................................................

III. READING: (25 points)

Part 3: Read the text and fill in each blank with ONE suitable word.
Our classes take place for three hours every morning from Monday to Friday. The maximum class size is
twelve (1) .................... the average is ten. We use modern methods of (2) .................... and learning, and
the school has a language laboratory, a video camera and recorders. You will only be successful in
improving (3) .................... English, however, if you work hard and (4) .................. speaking English as
much as you can. You will take a short (5) ................... in English as soon as you arrive. In this way, we
can put you in a (6) .................... at the most suitable level.
There are two classes at the Elementary level; one is for complete (7) ..................... and the other is
for students who know only a little English, in both classes you will practise simple conversations. In the
class (8) ..................... the intermediate level you will have a lot of practice in communication in real-life
situation because we help you to use the English you have previously (9) ................... in your own country,
You will also have the chance to improve your (10) ................... of English grammar and to build up your
vocabulary...
IV. WRITING (30 points)
Part 1: Rewrite each sentence so that it contains the word in capitals and so that the meaning stays the
same.
1. Why don't we go to the cinema tonight? FILM
Let's ......................................................................................................................................
2. One of my favourite pastimes is watching plays. THEATRE
Going ...................................................................................................................................
3. Americans are usually quite self-confident. STATES
People ...................................................................................................................................
4. John is still receiving treatment in the clinic. HOSPITAL
John.......................................................................................................................................
5. Children don't attend classes on Saturday. SCHOOL
Children don't .......................................................................................................................
6. Andrew said the mistake was my fault. BLAMED
Andrew..................................................................................................................................
7. It was raining all evening. STOP
It ...........................................................................................................................................

34
8. When he got to the party everyone was chatting and eating. ARRIVAL
On .................................................................................................................................... .....
9. They ate dinner and discussed the problem at the same time. WERE
While.....................................................................................................................................
10. Cats are not as loyal as dogs. MORE
Dogs .....................................................................................................................................
Part 2: COMPOSITION (You should spend about 30 minutes on this task.)
In about 250 words, describe a national festival or public holiday in our country, giving details of
the atmosphere as well as the events.

...............................................................................................................................................................
...............................................................................................................................................................
...............................................................................................................................................................
...............................................................................................................................................................
...............................................................................................................................................................
...............................................................................................................................................................
...............................................................................................................................................................
...............................................................................................................................................................
...............................................................................................................................................................
...............................................................................................................................................................
...............................................................................................................................................................
...............................................................................................................................................................
...............................................................................................................................................................
...............................................................................................................................................................
...............................................................................................................................................................
...............................................................................................................................................................
...............................................................................................................................................................
...............................................................................................................................................................
...............................................................................................................................................................
...............................................................................................................................................................
...............................................................................................................................................................
...............................................................................................................................................................
...............................................................................................................................................................
...............................................................................................................................................................
...............................................................................................................................................................
...............................................................................................................................................................
...............................................................................................................................................................
...............................................................................................................................................................
...............................................................................................................................................................
...............................................................................................................................................................
35
...............................................................................................................................................................
...............................................................................................................................................................
...............................................................................................................................................................
...............................................................................................................................................................
...............................................................................................................................................................
...............................................................................................................................................................
...............................................................................................................................................................
...............................................................................................................................................................
...............................................................................................................................................................

36
SỞ GIÁO DỤC VÀ ĐÀO TẠO KỲ THI CHỌN HỌC SINH GIỎI TỈNH
THỪA THIÊN - HUẾ LỚP 12 THPT NĂM HỌC 2004 - 2005
*** ------------------------------------------------------
ĐỀ CHÍNH THỨC Môn: TIẾNG ANH - (Vòng 2)
---------------------------------------------------------------------------------------------------------------------
HƯỚNG DẪN CHẤM

I. LISTENING: (20 points )


Part 1: (10 points): one point for each correct answer
1. I 2. D 3. A 4. E 5. H 6. B 7. C 8. F 9. G 10. K

Part 2: (10 points): one point for each correct answer.


Situation Problem Action offered Choice
1 broken zip on trousers refund
2 salty soup take it away
Choose something else
3 Keep them or send them on? send them on
4 watch has stopped new battery
5 repair and repaint it today accepts offer

III. VOCABULARY & GRAMMAR: (25 points )


Part 1. (10 points)

Q# 1 2 3 4 5 6 7 8 9 10
Key D C A D B C B A C A

Part 2. (10 points) one point for each correct answer


1. exclusively 2. comparative 3. determined 4. collectors 5. national
6. Deciding 7. observers 8. fascinating 9. usually 10. officially
Part 3 (5 points) one point for each correct answer
1. All students, except foreigners, must pay their fees to the university bursar.
2. Swimming is a sport I would recommend to any healthy person.
3. Addressing a large audience in the provincial capital yesterday, the president said that the new education
programme was essential.
4. Talks have begun between the representatives of the provincial assemblies to plan a railway linking the
provincial capitals.
5. The advertisement said that an expanding company wished to employ a secretary with good shorthand
and typing speeds at their head office.

37
IV. READING (25 points)
Part 2. (10 points) one point for each correct answer
1. and 3. your 5. test 7. beginners 9. learnt
2. teaching 4. practise 6. class 8. at 10. knowledge

V. WRITING: (30 points)

Part 1. (10 points) one point for each correct sentence


1. let's go to see/ and see a film.
2. Going to the theatre is something I really enjoy doing.
3. People in/ from the States are usually quite self-confident.
4. John is still in hospital receiving treatment.
5. Children don't go to school on Saturday.
6. Andrew blamed me for the mistake.
7. It did not stop raining all evening.
8. On his arrival at the party everyone was chatting and eating.
9. While they were eating dinner, they discussed the problem.
10. Dogs are more loyal than cats.

Part 2. (20 points)


The marks of the essay : 20
(Markers are requested to use the Marking Scheme for writing tasks)

Total marks : 100

---------------------------------------

38
M«n thi : TiÕng Anh
Thêi gian thi : 180 phót ( kh«ng kÓ thêi gian giao ®Ò)
Ngµy thi : 3/11/2008
§Ò thi gåm cã 9 trang

Sè ph¸ch
§iÓm cña bµi thi Hä, tªn vµ ch÷ kÝ cña gi¸m kh¶o (do chñ tÞch H§CT
ghi)
B»ng sè: Gi¸m kh¶o 1:

B»ng ch÷: Gi¸m kh¶o 2:

part I : Listening
Question I: You are going to hear an interview on transportation. As you listen, fill in the form below
with no more than three words or numbers:

Questions How do you How far is it How long Are you ever Suggestions
get to school ? from your does it take late because for improving
home to you to get to of transporta- the transpor-
school ? school ? tion problems station ?
?
Names

Mike By bus 3. 5. 7. 9.

Liz 1. 4. It depends No 10.

Tom 2. A few blocks 6. 8.

Question II: You are going to hear an interview on booking a holiday. Listen and tick() whether the
statements are True(T), False(F) or Not Given(NI):

Statements T F NI
1. They want to book a holiday for July.
2. They have decided where to go for the holiday.
3. Both customers are free to travel in the first week.

39
4. Last year, both of them visited France.
5. They would like to go to the mountains for skiing this year.
6. They don't want to go to Italy because the dates don't suit them.
7. They don't like to go to Sweden because there are no beaches.
8. It would be 385 pounds for them to visit Portugal
9. The customers prefer to visit Portugal by flight from London.
10. The flight stops at Manchester on the way to Portugal.

PART II : LEXICO - GRAMMAR:


Question I: Choose the word or phrase which best completes each sentence. Circle the letter A, B, C
or D next to the correct word or phrase.

1. We don't allow ................. in the classrooms.


A. people smoke B. smoke C. people to smoke D. to smoking
2. What a dangerous thing to do! You .................. have been killed!
A. may B. can ` C. must D. might
3. By the time I applied, all the holiday vouchers ................. used up.
A. had been B. have been C. were D. are
4. I felt as if I ................... a confidence.
A. have betrayed B. would betray C. had betrayed D. am betraying
5. The candidate ..................... to have withdrawn her application.
A. claimed B. said C. denied D. reported
6. How could you have .......................... him for your brother?
A. confused B. considered C. thought D. mistaken
7. It is vital that we .................... a change in people's attitudes.
A. bring down B. bring back C. bring about D. look after
8. We'll have to ................... down the options before coming to a decision.
A. slow B. narrow C. bring D. wind
9. The grocer's shop ................... since the supermarket opened.
A. closed B. has closed C. has been closing D. is closed
10. In ten year's time the population of the world...................... to 500 million.
A. grows B. is growing C. has grown D. will have grown
11. According to the rules of this contest, all employees and their immediate family member are
not fit to be chosen to enter the contest.
a. preferable b. qualified c. eligible d. desirable
12. When Sarah walked into the room and greeted everyone with a cheery “ hello” , it seemed as
if all the gloom in that place was driven out.
a. expelled B. ejected c. banished d. removed
13. When we heard the unexpected news, we were at a loss for words.
a. could not speak for a while b. did not know what to say
c. became dumb d. panicked

40
14. “ Mum, could you please help me with my homework?”
“ Could you ask your father? I have my hands full at the moment.”
a. am very busy b. am very tired
c. have to go out d. have a lot on my mind
15. “ Who can tell me what man is? Is he a carnivore or a plant-eating animal or both?” The
science teacher asked the class.
a. omnivore b. predator c. herbivore D. equator
Your answers:
1……….. 2…………. 3………… 4………… 5……………
6……….. 7…………. 8………… 9………… 10…………..
11……… 12………… 13………. 14………. 15…………..
Question II : Give the correct tense and form of the verb to fit each gap.
1. It was essential that the union (1.moderate)……………its demand for a shorter working week.
2. Were I (2. know)…………….. the answer, I would tell you right away.
3. It was our fault to keep Mary waiting so long. We (3.inform)……………her in advance.
4. For the past few days, I (4.work)……………in Jack's office, as my own office
( 5. redecorate)………….
5. I wish I (6.know)……………you needed the book. I (7.buy)……………it for you in London.
6. Too big and too heavy (8.pull)…………..behind passenger cars, these mobile homes are
moved by tow trucks.
7. The Amazon valley is extremely important to the ecology of the earth. Forty percent of the
world's oxygen ( 9. produce)…………. there.
8. I was such a beautiful child that my parents used to have me (10. kidnap)………..just to see
my picture in the newspapers.

Your answers:
1……….. 2…………. 3………… 4………… 5……………
6……….. 7…………. 8………… 9………… 10…………..
Question III : Read the passage and fill in each blank with the correct form of the word
provided

Modern advertisements contain hidden messages. (1) IMPLY in the advertisements showing the
pretty girl in the new car or the smiling children round the packet of washing powder is the message that if
we buy the (2) PRODUCE , we also achieve success and happiness. It is a subtle approach since it seeks
to exploit our secret teams, and it is (3) ESCAPE since advertising is ubiquitous, giant street hoardings and
(4) CATCH jingles on TV bombard us form all sides. They brainwash us into believing that we can realize
our ambition quickly and (5) EASE .On the other hand, defenders of advertising say that it is (6)
BENEFIT. Advertising is (7) INFORM Advertisements tell us about useful new products. They (8)
BRIGHT our lives with colour and music. They increase demand, stimulate industry and so keep prices
down. Whether for or against advertising, most people would agree that some kind of watch-dog body, (9)

41
APPOINTMENT by government or by the advertising industry itself, necessary to maintain standards or
honesty and to discourage the more blatant types of (10) LEAD advertisements.

Your answers:
1……….. 2…………. 3………… 4………… 5……………
6……….. 7…………. 8………… 9………… 10…………..
Question IV : There are ten mistakes in the following passage. Underline the mistakes, number and
correct them.
Priscilla Presley, actress and widower of Elvis Presley, entered the acting profession quiet accidentally.
On being asked to do a television commerce for a well-known shampoo, she took which turned out to be an
important decision: to take actor classes.
She did this in an attempt to overcome her acute shyness at being in the public eye, but to her
amazement she loved every minutes. Her extreme lack of confidence stemmed from the early days of her
marriage with Elvis, which she remembers were often spent sitting in dark, depressed hotel rooms away
from the glare of publicity, who Elvis was so anxious to avoid.
Film work, including the high successful slapstick comedy The naked Gun, quickly followed the
television contract.

Your answers:
1……….. 2…………. 3………… 4………… 5……………
6……….. 7…………. 8………… 9………… 10…………..

Question V: Read the following passage and use ONE word to fill in each gap.
The Great (1)………………of Giza was built as a (2)………..of Pharaoh Cheops in 2720 BC.
(3)……………..it was built a long time ago, its (4)………………….makes it one of the true
(5)…………..of the world. The four (6)…………….of the pyramid are put almost exactly on true north,
south, east and west.
Explorations and detailed examinations of the base of the structure reveal many interesting lines.
Further (7)……….. study indicates that (8)…………..represent a type of line of events-past, present and
future. Many of the (9)………….have been interpreted and found to coincide with known facts of the past.
Was this super structure made by ordinary beings or one built by a race (10)…………superior to
any known today?
Your answers:
1……….. 2…………. 3………… 4………… 5……………
6……….. 7…………. 8………… 9………… 10…………..
part III : READING.
Question I . Read the passage carefully and then choose the best answer to each sentence.
Hundreds of thousands of persons each year fall prey to some type of cancer, but new methods of
radiation therapy have enabled doctors to save more lives than ever before. Medical researchers have

42
developed several experimental forms of this time –honored cancer treatment that seem effective in
fighting the disease.
One promising approach involves exposing cancer cells to radiation by implanting a radioactive
source directly into the malignant tissue. This process greatly increases the dosage and thus the
effectiveness of the treatment. Another technique utilizes drugs to make cancer cells more susceptible to
the effects of radiation and to make normal cells more resistant. Certain drugs are able to neutralize the
genetic framework of cancer cells, thus making them more easily affected by radiation. Both techniques
have seen some positive results in the treatment of inoperable brain tumors.
These and other methods have helped to raised the recovery rate for cancer victims from 30 percent
40 years ago to around 50 percent today. This is encouraging news for those who fall prey to one of the
world’s leading killers.
1. What is the author’s main purpose in the passage ?
A. To provide statistical information on cancer.
B. To argue for new methods of cancer treatment.
C. To illustrate new techniques of radiation therapy.
D. To give the results of recent cancer research.
2. According to the passage, which of the following is true about radiation therapy?
A. There is only one effective form of this therapy.
B. It saves millions of lives each year.
C. It is an accepted method of cancer treatment.
D. It causes the incidence of cancer to raise dramatically.
3. In paragraph 2, the word “ malignant ” most closely means :
A. diseased B. experimental C. treated D. porous
4. According to the passage, radiation therapy is most effective when :
A. Drugs are used to relax the cancer patient.
B . The cancer is directly exposed to the radioactive material.
C. It is used on as many patients as possible.
D. The cancer cells are resistant to treatment.
5. It can be inferred from the passage that :
A. Improvements in cancer treatment during the last half century have been relatively
ineffective.
B. The number of deaths caused by cancer has decreased substantially.
C. Fewer people are susceptible to the effects of cancer.
D. Scientists are close to eliminating cancer entirely.
Your answers:
1……….. 2…………. 3………… 4………… 5……………

Question II. For question from 1 – 5, read the following text carefully and then choose from the list
A- I the best phrase given below to fill each of the spaces. Each correct phrase may only be used
once. Some of the suggested answers do not fit at all. The exercise begins with an example ( 0 ).
MODERN ENGLISH

43
Imperceptibly, during the 18th century, English loses the most noticeable remaining features of structural
difference ( 0 ).......I..........By the end of that century, with but a few exceptions, the spelling, punctuation,
and grammar are very close to ( 1 ).................If we take an essay by William Hazlett ( 1775 – 1830 ) or a
novel of Jane Austen ( 1775 – 1817 ), for example, we can read for pages ( 2 )...................We would find
the vocabulary somewhat unfamiliar in places, the idiom occasionally unusual or old-fashioned, the style
elegant or quaint, and we might feel that the language was in some indefinable way characteristic of a
previous age : but we do not need to consult a special edition or historical dictionary at every turn ( 3
).............Jane Austen makes demands on our modern linguistic intuitions which seem little different from
those required by Catherine Cookson or PD James.
However, despite this apparent continuity, the language at the end of the 18th century is by no
means identical to what we find today. Many words, though spelt the same, had a different meaning.
( 4 )...............we would also notice several differences in pronunciation, especially in the way words were
stressed. And an uninformed modern intuition would achieve only a superficial reading of literary texts of
the period. In reading a novel of the 1990s, we can make an immediate linguistic response to the social and
stylistic nuances introduced into the text, ( 5 )...............we recognise the differences between formality and
informality, or educated and uneducated; we can sense when someone is being jocular, ironic, risquÐ,
archaic or insincere. We can easily miss such nuances in the writing of the early 19 th century, especially in
those works which take the manners of contemporary society as their subject. That world is more
linguistically removed from us than at first it may appear.
A. in order to understand the text.
B. that deceive in its apparent continuity.
C. because the context often enables us to see the intended sense.
D. what they are today
E. because we are part of its age
F. when we know it had an additional meaning at that time
G. if we had tape recordings of the time
H. before a point of linguistic difference might make us pause
I. which distance the Early Modern English period from us

Your answers:
1……….. 2…………. 3………… 4………… 5……………

Question III. Read the following text and decide which word best fits each blank.

OSCAR’S WINNING PERFORMANCE


Two boats, engines paralysed are drifting helplessly towards rocks in a raging sea. Gale-force winds
are blowing as a distress message is relayed to the (1 )...............The west coast search – and- rescue
helicopter takes off from Shannon; its destination is Clew Bay in County Mayo.
The terrified crews on Sundancer and Healther Berry are only half a mile from disaster when Hotel
Oscar, the Irish Marine Emergency Service helicopter arrives and the winch crew
( 2 ).............saving their lives. There is no hope for the boats – the conditions are too bad for that. The
threatening rocks will make matchwood of them.

44
It’s not easy to get the rescue line down on the pitching, rolling decks as the pilot, Captain Al
Lockey hovers directly ( 3 )...............By the time the exhausted winchman has picked up the two crew
members of Healther Berry , the helicopter is running ( 4 )........................on fuel. The pair on Sundancer
will have to be abandoned if ( 5 ).....................else is to survive. As if that decision isn’t difficult enough,
screaming winds make for a treacherous flight out of the bay.
For Captain Locky, 25 years a helicopter pilot and veteran of typhoon conditions off oil rigs in the
South China Sea, this was the worst experience in a distinguished ( 6 )...................In fact, a change in wind
direction was to spare. Sundancer its horrible face, much to the ( 7 ).............of the rescue crew whose
hearts were breaking as they were forced to turn their backs and ( 8 )............for home. Medals, it is said
should be given to those who have to make that most painful decision to say “ no ”. Fortunately, most
crews can and (9 )...............say “ yes ” in all conditions and at all
( 10 )..............of night and day.
That was Mission 47, accomplished just over three months after Hotel Oscar’s contract began in
July 1991.
1. A. shore B. land C. beach D. seaside
2. A. set off B. set up C. set out D. set about
3. A. above B. higher C. ahead D. over.
` 4. A. low B. clown C. short D. out
5. A. no one B. everyone C. someone D. all
6. A. job B. role C. profession D. career
7. A. satisfaction B. comfort C. relief D. gratitude
8. A. go B. fly C. head D. lend
9. A. should B. do C. may D. need
10. A. periods B. moments C. hours D. minutes

Your answers:
1……….. 2…………. 3………… 4………… 5……………
6……….. 7…………. 8………… 9………... 10…………..
D. Writing
Question I : Rewrite these sentences so that the new one has a similar meaning as the sentence
preceded:
1. He did not pass his driving test until he was nearly 30 years old.
It ..........................................................................................................................................
2. He is determined to campaign for his son’s freedom.
He has no............................................................................................................................
3. Stock brokers bought the company shares because they assumed that it was making a lot of profit.
Had .....................................................................................................................................
4. The financial crisis affects both developed and third-world countries.
Not only ..............................................................................................................................
5. He was very sorry because he didn’t have time to visit all places of interest in London when he was
there last year.
He greatly ...........................................................................................................................
45
Question II: Do not change the given word, use it to rewrite each of the following sentences so that
each has as a similar meaning as the original one.
1. Although he had his legs cut off after a serious accident, he competed in the Olympic Games and won a
medal. (Prevent)
............................................................................................................................................
2. They suddenly realized that they were on the verge of a bankrupt. (Dawned)
............................................................................................................................................
3. The rocketing prices have proved too much for most salaried people. (Cope)
............................................................................................................................................
4. We must accept the fact that we haven't made much progress in the struggle against poverty. (Terms)
............................................................................................................................................
5. Illiteracy has been one of the causes of poverty.( Contributed)
............................................................................................................................................
Question III: Chart description:
The bar chart compares the prices of hotels and restaurants in 10 big cities in the world.
Write a report, describing the information shown below.
You should write at least 150 words.
Restaurant and hotel prices

400 375 382

350 318
307
300 261
250 232 225 236
219
200
146
150

100 72
52 44
50 25 23 22 31 22
10 10
0
Budapest Frankurt Hong London Madrid Milan New York Paris Prague Tokyo
Kong

$ Hotel $ Restaurant

46
N.B. Price of a double room with bath, incl. breakfast and service, in a 1st class international hotel.
Price of a dinner for one (rib or sirloin steak with two side dishes and a dessert; without drinks), incl.
service, in a good restaurant.

............................................................................................................................................................................
............................................................................................................................................................................
............................................................................................................................................................................
........................................................................
............................................................................................................................................................................
..........................................................................................................................
............................................................................................................................................................................
..........................................................................................................................
............................................................................................................................................................................
..........................................................................................................................
............................................................................................................................................................................
..........................................................................................................................
............................................................................................................................................................................
..........................................................................................................................
............................................................................................................................................................................
..........................................................................................................................
............................................................................................................................................................................
..........................................................................................................................
...................................................................................................................................................
............................................................................................................................................................................
..........................................................................................................................
............................................................................................................................................................................
..........................................................................................................................
............................................................................................................................................................................
..........................................................................................................................
............................................................................................................................................................................
..........................................................................................................................
...................................................................................................................................................

Question IV: Essay writing:


It is very important that children should study hard at school. Time spent playing sports and having fun
is time wasted. Do you agree?
Use your own ideas, knowledge and experience and support your arguments with examples and relevant
evidence.
You should write at least 250 words.

47
............................................................................................................................................................................
............................................................................................................................................................................
............................................................................................................................................................................
........................................................................
............................................................................................................................................................................
..........................................................................................................................
............................................................................................................................................................................
..........................................................................................................................
............................................................................................................................................................................
..........................................................................................................................
............................................................................................................................................................................
..........................................................................................................................
............................................................................................................................................................................
..........................................................................................................................
............................................................................................................................................................................
..........................................................................................................................
............................................................................................................................................................................
..........................................................................................................................
............................................................................................................................................................................
..........................................................................................................................
............................................................................................................................................................................
..........................................................................................................................
............................................................................................................................................................................
..........................................................................................................................
............................................................................................................................................................................
..........................................................................................................................
............................................................................................................................................................................
..........................................................................................................................
............................................................................................................................................................................
..........................................................................................................................
............................................................................................................................................................................
..........................................................................................................................
............................................................................................................................................................................
..........................................................................................................................
............................................................................................................................................................................
..........................................................................................................................

r-êng thpt chuyªn


nguyÔn tÊt thµnh

H-íng dÉn chÊm ®Ò chÝnh thøc


Chän häc sinh giái m«n tiÕng Anh n¨m häc 2008 - 2009

48
Part I : LISTENING (3)
Question I: (10 x 0.1)
1. by car 2. by bike 3. 5 miles 4. 20 miles 5. 15-20 minutes
6. 15 minutes 7. Yes 8. No 9. Need more buses 10. Better subway system
Question II : (10 x 0.2 )
1. T 2. F 3. F 4.T 5. NI
6. T 7. NI 8. T 9. F 10. NI

PART II : LEXICO - GRAMMAR: (6.5)


Question I(15 x 0.1 )
1.C 2.D 3.A 4.C 5.A
6.D 7.C 8.B 9.B 10.D
11.B 12.D 13.B 14.A 15.C
Question II: (10 x 0.2 )
1. moderate/ should moderate 2. to know 3. should have informed
4. have been working 5. is being redecorated 6. had known
7. could/ would have bought 8. to be pulled 9. is produced
10. kidnapped

Question III: (10 x 0.1 )


1. implication 2. product 3. inescapable 4. catchy 5. easily
6. beneficial 7. informative 8. brighten 9. appointed 10. misleading

Question IV: (10 x 0.1 )


1. widower  widow
2. quiet quite
3. which what
4. commerce commercial
5. actor acting
6. minutes minute
7. with to
8. depressed depressing
9. who which
10. high highly

Question V: (10 x 0.1 )


1. pyramid 2. tomb 3. Although 4. construction 5. wonders 6. sides
7. scientific 8. these 9. events 10. far

49
Part III : READING (4)
Question I . (5 x 0.2)
1. C 2. C 3. A 4. B 5. B
Question II. (5 x 0.2)
1.D 2.H 3. A 4.G 5.E

Question III. (10 x 0.2)


1.A 2.D 3.A 4.A 5.B
6.D 7.C 8.C 9.B 10.C

part IV: Writing(6.5)


Question I : (5 x 0.2)
1. It was not until he was nearly 30 that he passed his driving test.
2. He has no intention to stop campaigning for his son’s freedom.
3. Had stock brokers knew that the company was not making profit they wouldn’t have invested in
it/bought its shares.
4. Not only are developed countries affected but also third-world countries in the financial crisis.
5. He greatly regretted that he didn’t have time / not having (enough) time / to visit all places of
interest in London when he was there last year.
Question II: (5 x 0.2)
1. His disability / lame/ legs being cut off did not/could not prevent him from competing in the
Olympic Games and winning a medal.
2. It suddenly dawned on them that they were on the verge of a bankrupt.
3. Most salaried people haven’t been able /cannot cope with the rocketing prices.
4. We must come to terms with the fact that we have not made much progress in the struggle against
poverty.
5. Illiteracy has contributed to poverty
Question III: Chart description: (1,5)
 Style: appropriate: 0.25
 Spelling + Grammar: accurate: 0.5
 Answer the question asked: 0.25
 Variety of appropriate language: 0.25
 Coherence: 0.25
Question IV: Easy writing: (3 )

50

You might also like